subject
Mathematics, 29.03.2021 21:40 alexandergonzalez38

PLEASE ANSWER HONESTLY (PLEASE BE HELPFULL I HAVE BEEN STUCK ON THIS PROBLEM) WILL MARK BRANIEST IF CORRECT. (no links my computer is slow and no ctrl c ctrol v if possible Given a right triangle with leg lengths f and g, and hypotenuse h, if f = 7 cm and h = 11.2 cm, what is g?

Show your work.

ansver
Answers: 3

Another question on Mathematics

question
Mathematics, 21.06.2019 16:00
Atrucker had a load of grain containing 2 tons. she unloaded 1 ton and 1,200 pounds at the warehouse. how much grain does she still have left on the truck?
Answers: 1
question
Mathematics, 21.06.2019 18:30
If the value of x increases by 5, how does the value of m(x + 3) change? a) it increases by 5m. b) it increases by 8m. c) it decreases by 5m. d) it decreases by 8m.
Answers: 1
question
Mathematics, 21.06.2019 23:00
Agroup of three friends order a combo meal which includes two slices of pizza and a drink each drink is priced at $2.50 the combined total of $31.50 how much is the cost of a slice of pizza
Answers: 2
question
Mathematics, 22.06.2019 00:40
B. two events are dependent if the occurrence of one event changes to occurrence of the second event. true or false
Answers: 2
You know the right answer?
PLEASE ANSWER HONESTLY (PLEASE BE HELPFULL I HAVE BEEN STUCK ON THIS PROBLEM) WILL MARK BRANIEST IF...
Questions
question
Mathematics, 01.03.2020 05:48
question
English, 01.03.2020 05:49
question
English, 01.03.2020 05:50
question
Mathematics, 01.03.2020 05:52
question
Mathematics, 01.03.2020 05:53
question
English, 01.03.2020 05:54
question
English, 01.03.2020 05:58
question
Mathematics, 01.03.2020 05:59
Questions on the website: 13722363